k Bestimmen,dass Graph mit der 1. achse eine fläche vom angegebenen Flächeninhalt einschließt

Neue Frage »

telefonites Auf diesen Beitrag antworten »
k Bestimmen,dass Graph mit der 1. achse eine fläche vom angegebenen Flächeninhalt einschließt
hallo, haben hier einen batzen aufgaben bekommen die man alle nach dem selben schema ausrechnen kann.
leider hab ich absolut keinen schimmer und bräuchte vielleicht eine kleine vorrechnung, die anderen 5 mache ich dann komplett selbst.
weiß nur überhaupt nicht wie ich anfangen soll :
Bestimmt k so, dass der Graph der Funktion f mit der 1. Achse eine Fläche vom angegebenen Flächeninhalt A einschließt.

gegeben ist f(x)= x^2- k^2 und A= 10

kann mir jemand weiterhelfen? warscheinlich ist das total einfach, aber mir fehlen leider anfangs immer die gedankeschübe!
Lazarus Auf diesen Beitrag antworten »

der graph ist eine verschobene normalparabel, d.h. die stammfunktionen sind alles kubische parabeln.

um die fläche zu bestimmen musst du folglich erstmal die nullstellen bestimmen (da nach den nullstellen die fläche unendlich groß wird.)

diese nullstelle wird in abhängikeit von k erscheinen, und bildet somit deine obere grenze.

wenn du nun integrierst, dann bekommst du eine funktion
und da du weisst das diese den wert 10 haben soll setzt du das ein, löst nach k auf und fertig!

system durchschaut ???

servus
telefonites Auf diesen Beitrag antworten »

nein, leider nicht.
häufig ist mathe für mich wie chinesisch, noch hab ich aber zeit es zu kapieren. wärest du so lieb und rechnest mir einmal so eine aufgabe vor damit ich die anderen alleine lösen kann?
mfg, telefonites
JochenX Auf diesen Beitrag antworten »

also hier noch mal als rezept, du sagst dann genau, was du warum nicht verstehst


1) Nullstellen in abhänigkeit von k bestimmen
2) allgemeine Parabel integrieren (stammfunktion suchen) mit denoben gefundenen nullstellen als grenzen

ergibt dir integral (flächeninhalt) in abhängigkeit von k, also eine flächenfunktion

3) diese k-abhängige formel =10 setzen und berechnen, für welche(s) k du fläche 10 hast

mfg jochen
Lazarus Auf diesen Beitrag antworten »

es gibt keineKomplettlösungen hier!

der lösungsweg ist genau so wie ich oder loed in kurz ( Augenzwinkern ) zusammengefasst hat, und vorallem ist er für aufgaben dieser art immer gleich!

wenn ich eine fläche zwischen irgnedwas berechnen muss, immer erst schnittpunkt bzw nullstellen ausrechnen.

wenn ich eine funktionenschar hab, bei der die nullstelle(n) abhänig von k sind, bekomm ich nach dem integrieren immer eine flächenfunktion in abhänigkeit von k.

wenn das ergebnis eines integral bekannt ist, ist immer eine grenze unbekannt (bzw eine unbekannte benutzt als grenzenangabe). Nach diese muss dann aufgelöst werden.

und jetzt versuch doch mal bitte anhand des lösungschemas (siehe beitrag von loed) diese aufgabe zu bearbeiten, und falls nötig schildere deine probleme!

ciao
ltelefonites Auf diesen Beitrag antworten »

ne, ich rall das immer noch nicht.
also wie komm ich denn auf die nullstellen? und die dienen mir dann als ober und untergrenze bei der stammfunktion oder wie?
stammfunktion wär demnach 1/3*x^3-k^2*x
und dann hab ich die nullstellen und stammfunktion und dann?
also dann (1/3*(obere grenze)^3-k^2*(obere grenze))- ((1/3*(untere Grenze)^3-k^2*(untere Grenze))= 10 ..dann nach k aufkösen?
ich verstehe es wirklich nicht!
 
 
telefonites Auf diesen Beitrag antworten »

ist dann die Nullstelle die wurzel aus k^2?
derkoch Auf diesen Beitrag antworten »

Zitat:
also wie komm ich denn auf die nullstellen? und die dienen mir dann als ober und untergrenze bei der stammfunktion oder wie?


wie berechnest du sonst die nullstellen?
sicher sind das deine integrationsgrenzen.
telefonites Auf diesen Beitrag antworten »

oder sowas :
10 = 1/2* (Wurzel aus k^2)^3-k^2*Wurzel aus k^2
derkoch Auf diesen Beitrag antworten »

Zitat:
1/2* (Wurzel aus k^2)^3-k^2*Wurzel aus k^2


wo kommt das denn her?


was ist denn ??
telefonites Auf diesen Beitrag antworten »

ich hab die funktion gleich 0 gesetzt und dann kam als nullstelle wurzel aus k^2..das ist dann ja die obere grenze..und was die untere?
telefonites Auf diesen Beitrag antworten »

und das ist die stammfunktion eingestezt mit der oberen grenze (wurzel als k^2) und diese müsste dann ja =10 sein?!?
man ich rall das nicht.....
derkoch Auf diesen Beitrag antworten »

Zitat:
Original von Lazarus

wenn du nun integrierst, dann bekommst du eine funktion
und da du weisst das diese den wert 10 haben soll setzt du das ein, löst nach k auf und fertig!

servus



anscheinend bist du über diesen tipp nur rübergeflogen ohne dir gedanken zu machen! die lösung steht schon fast da!
telefonites Auf diesen Beitrag antworten »

nein, bin ich nicht!
hab mit sogar das sogenannte ''rezept'' abgeschrieben und es in meine mappe geheftet. leider versteh ich es offensichtlich nicht.
aufgrund dessen hab ich auch keine ahung wie ich dann auch die nullstellen komme. habe die funktion gleich 0 gesezt und da kam : wurzel aus k^2 raus.....
auch weiß ich nicht ob ich dann 10 = F (mit der obergrenze)-F(mit der untergrenze) rechnen muss oder einfach nur 10 = F (mit der Obergrenze, die in dem fall ja k ist (wo ich immer noch nicht weiß wie man drauf kommt!!!)und dann nach k auflöse!!!!!!!!!!!!
derkoch Auf diesen Beitrag antworten »



du wirst mir doch nicht erzählen wollen, daß du das hier nicht auflösen kannst?
telefonites Auf diesen Beitrag antworten »

doch, das will ich dir erzählen. weil für mich k einfach eine normale zahl ist wie z.b. 2 und 2^2 ist4 und wegen dem x^2 muss ich davon die wurzel ziehen deshalb : die wurzel aus k^2
derkoch Auf diesen Beitrag antworten »

und was ist die wurzel aus 4?
telefonites Auf diesen Beitrag antworten »

dann lieg ich mit meinem worzel aus k^2 nicht falsch.....ist ja auch egal
aber was dann?
ist dann meine untere grenze immer null?
wie gehts wieter?
derkoch Auf diesen Beitrag antworten »

du schleppst immer dein blödes mit dir rum, dadurch kommst du vielleicht ins stolpern!



da hast du die grenze, wie lazarus dir schon hingeschrieben hat!
telefonites Auf diesen Beitrag antworten »

ja und dann? kann man mir nicht weiterhelfen?
was ist mit der unteren grenze, was mach ich dann weiter?
Lazarus Auf diesen Beitrag antworten »

*gg*

ich dachte das wäre das geringste problem, daher hab ich des sozusagen als gegeben gleich mit angegeben.

tut mir leid, wenn des zu verwirrung geführt hat.

warum man die null als untere grenze wählt is hoffentlich auch klar:
es geht um die fläche im ersten quadranten.
und 0 ist da des linkestes was möglich ist (von der anschauung her ...)

machst du weiter?

servus
telefonites Auf diesen Beitrag antworten »

naja hab mir dann gedacht mit der Integralfunktion weiterzurechnen:
10 = 1/3*k^3-k^2*k / : 1/3
30 = .....
aber irgednwie ist das quatsch unglücklich
derkoch Auf diesen Beitrag antworten »

fasse die rechte seite zusammenbevor du teilst!
du bist auf dem richtigen weg!
telefonites Auf diesen Beitrag antworten »

und was ist ''ein erster quadrant''? ohoh wie peinlich traurig
telefonites Auf diesen Beitrag antworten »

da ja punkts vor strichrechnung ist müsste auf der rechten siete stehen : 1/3k^3-k^3.....wenn ich dann aber duch 1/3 teile hab ich auf der rechten seine nur noch k^3-k^3 stehen und das wär ja NULL verwirrt
derkoch Auf diesen Beitrag antworten »

Zitat:
Original von telefonites
und was ist ''ein erster quadrant''? ohoh wie peinlich traurig


ach du dickes ei! ist doch nicht ernst gemeint oder?

http://www.mathe-online.at/mathint/zeich/i.html
telefonites Auf diesen Beitrag antworten »

ich meine alles erst was ich schreibe...sehe das zum ersten mal und hab es mittlwerweile in die 12e klasse geschafft, sogar gar nicht mal schlecht in mathe unglücklich
derkoch Auf diesen Beitrag antworten »



erstmal zusammenfassen!
telefonites Auf diesen Beitrag antworten »

also ich nehme mal an das da so ganz warscheinlich 1/3 rauskommt.
logisch wär aber für mich (wegen punkt vor strich) das obere?!?
telefonites Auf diesen Beitrag antworten »

ich weiß es doch wirklich nicht !!!!!!! traurig
derkoch Auf diesen Beitrag antworten »

Zitat:
Original von telefonites
also ich nehme mal an das da so ganz warscheinlich 1/3 rauskommt.
logisch wär aber für mich (wegen punkt vor strich) das obere?!?


nein! nix punkt vor strich!



jetzt ist nix mehr mit punkt und strich!

BRUCHRECHNUNG!
telefonites Auf diesen Beitrag antworten »

ja gut, kann ich nachvollziehen.bringt mich aber auch nicht wieter
10=k^3/3 - k^3 dann mal 3.....dann komme ich letztendich wieder auf die 0 von vorhin
mercany Auf diesen Beitrag antworten »

Zitat:
Original von telefonites
ja gut, kann ich nachvollziehen.bringt mich aber auch nicht wieter
10=k^3/3 - k^3 dann mal 3.....dann komme ich letztendich wieder auf die 0 von vorhin


Warum kommst du da bitte auf Null?!?!?!

Zeig dochmal, wie du das zusammengefasst hast - würde mich mal interessieren! smile
Neue Frage »
Antworten »



Verwandte Themen

Die Beliebtesten »
Die Größten »
Die Neuesten »